PT35.S1.Q15 - statistician: a financial magazine claimed that its survey of its

rogersalexandra7rogersalexandra7 Alum Member
edited May 2017 in Logical Reasoning 213 karma

I chose C because it is weakening the argument which is saying that people are more concerned about their finances than politics and C says that they are JUST AS concerned about their finances and politics thus weakening the argument and I thought E was incorrect because 1) it is bringing outside information that wasn't mentioned in the stimulus and 2) it says that they are concerned with politics and their finances but it doesn't say to what degree. I mean the conclusion is saying that they are concerned about their politics but the degree of concern is different.

Can someone explain this to me? Thank you!!!!
https://7sage.com/lsat_explanations/lsat-35-section-1-question-15/

Comments

  • BinghamtonDaveBinghamtonDave Alum Member 🍌🍌
    8699 karma

    I believe you have misidentified the specific question you are referencing. The question beginning: "public health experts have waged." Is not what you wrote about in your post. Please edit accordingly with proper location of the problem and I will follow up with a response about the problem to the best of my abilities.

    David

  • inactiveinactive Alum Member
    12637 karma

    Please also include the section# so we know what section you're talking about.

  • rogersalexandra7rogersalexandra7 Alum Member
    213 karma

    sorry! its section #1

  • inactiveinactive Alum Member
    12637 karma

    Edited title! Bumping to the top so people can see.

  • rogersalexandra7rogersalexandra7 Alum Member
    213 karma

    Thank you!

  • 44 karma

    One thing to keep in mind and to differentiate are the two arguments. One is the statistician's argument and the other is the one being made by the financial magazine.

    For answer choice C: It weakens the financial magazine's argument by saying that other surveys suggest otherwise. This strengthens the statisticians argument so would not be a candidate for a strengthen EXCEPT question. If you follow JY's procedure you would circle and move on (circle 4 strengthen and the 5th is your correct answer).

    For answer choice E let's first look at the Question stem that reads "Each of the following, IF TRUE." So we accept the answer choices, in the context of a strengthen answer choice it would be hard to strengthen the argument with only the argument itself so outside information isn't a negative. If you look at answer choice C which I think you originally selected, "other surveys," also contains outside information from the stimulus. As for why E is correct, it does nothing to assist the Statistician's argument. It's blah. Okay so they are concerned with other things, but to what degree? Who knows? Therefore it doesn't strengthen. It doesn't have to weaken, just NOT strengthen. That's what we have here. Hope that helps.

  • rogersalexandra7rogersalexandra7 Alum Member
    213 karma

    thank you!

Sign In or Register to comment.